Download as docx, pdf, or txt
Download as docx, pdf, or txt
You are on page 1of 86

Reading Comprehension

In the mid-fifties when GE was expanding its operations across the globe, the president of GE, Ralph
Cordiner decided to set up a corporate university an hour away from New York spread over 59 acres. In
1956 GE offered its first course that spread over 13 weeks. Today no executive can imagine spending a
full quarter of the year sitting in a classroom. While the courses that are offered at Crotonville have
become shorter, the efficacy of the investment remains unquestionable. The headhunters refer to GE as
a leadership factory.

GE is not alone. McDonalds set up its Hamburger University in 1961. When Steve Jobs hired Joe Podolny,
the then dean of Yale to start Apple University in 2008 it made a big splash. Apple University drew
faculty members including professors from universities like Yale, Harvard, Berkeley, Stanford and MIT.

The trend of corporate universities is on the rise. The companies have a large employee base that ranges
from 8,000 to 300,000 employees or more. They cut across sectors and businesses from automobiles to
pharma and everything else in between. General Motors Institute, Caterpillar, Unilever, GDF Suez,
Veolia, Axa, Sanofi, Novartis, Petronas, and many more.

In 1993, corporate universities existed in only 400 companies. By 2001, this number had increased to
2,000, including Walt Disney, Boeing, and Motorola. According to BCG, there are estimated to be more
than 4,000 companies with formal corporate universities.

Scale: McDonalds serves 68 million customers daily in 119 countries across 35,000 outlets. This
complexity requires training. That ensures that McDonalds burgers taste the same from Mumbai to
Moscow.

Complexity: When surgeons move to the operating theater they rely on checklists. These lists
reduce human error, and help the surgeon to operate with speed without having to stop and double
check if they are missing any step. In performing complex tasks through collaborating teams, speed
comes through training in standard processes and procedures. Apple University teaches employees that
they’re at the company to be the very best at one specific task.

Corporate Governance: Governance needs the ability to manage reporting relationships, finances, and
facilities. Compliance and risk management demand investments in training. On any given day two
billion people use Unilever products across countries that have different rules and regulations. This scale
and complexity demands constant investment to ensure standards are not compromised.

Education: Entry-level hires are drawn from a wide variety of educational backgrounds. There is a need
to bring people to a common minimum level of knowledge and skills. From technical skills or domain
knowledge or personal competencies, people need to keepupgrading.

Values and Culture: The leaders need to be role models of the values the company proclaims. When the
majority of employee responses to everyday situations become uniform, it forms the culture of the
organization. Tying the professional development to strategic challenges is a strong reason to invest in
training.
The culture of the organization has to support its vision and strategy. This needs a place when people
can come together and connect with others and learn. Technical training can be delivered through e-
learning modules. It is the intangibles like culture that people need to learn from role models.

What does Apple University teach the employees of Apple?

a. That they need to have superior communication skills.

b. That since they work at one of the biggest brands of the world, they cannot afford to make an error.

c. That they need to master every skill.

d. That they need to be the very best in the work they do.

e. That the importance of the task assigned to them cannot be stressed enough

ANSWER: That they need to be the very best in the work they do.

Explanation:
Check para 6, last sentence

What, according to the author, do you think is the main challenge of increasing scale?

a. The main challenge is to engage an increasing number of employees for handling the scaled up
operations.

b. The main challenge is to break down the complex operation into simple steps.

c. The main challenge is to train the employees involved in the large scale operations.

d. The main challenge is to maintain a consistency of output quality.

e. Not clearly mentioned.

ANSWER: The main challenge is to maintain a consistency of output quality.

Explanation:
Check para 5, sentence 3
See para 3, sentence 1 & 2

In the context of the passage, choose the word which is most opposite in meaning to the given word.

Intangible

a. Priceless

b. Ambivalent

c. Transparent
d. Cheap

e. Definite

ANSWER: Definite

Explanation:
Eg: There is a quality about her that draws me to her; it is something intangible, something I cannot
put a finger on.

In the context of the passage, choose the word which is most opposite in meaning to the given word.

Upgrade

a. Deteriorate

b. Elevate

c. Learn

d. Decentralize

ANSWER: Deteriorate

Explanation:
Eg: There is so much competition nowadays that even doctors need to keep upgrading, i.e., learning
new methods of treatment.

Choose the word/ phrase which is most similar in meaning to the given word.

Human error

a. The flawed logic of comparing human beings to machines

b. A term coined by psychologists meaning ‘deviation in human thought’

c. The propensity for making mistakes as a result of being human

d. Error in recognizing human potential

NSWER: The propensity for making mistakes as a result of being human

Explanation:
Eg: Self-driving cars by Google aim to reduce road accidents by eliminating scope for human error.
Since these run on technology and no human driver is required, there is no scope for human error.

Choose the word/ phrase which is most similar in meaning to the given word.
Efficacy

a. Effectiveness

b. Monetary returns

c. Need

d. Futility

e. Tenure

ANSWER: Effectiveness

Explanation:
For Eg: The efficacy of the medicine, i.e., its ability to cure the patient, has been proved over the trial
period.

According to the author, what could be a good reason to invest in the training

a. Strategic growth can be brought about by professional development.

b. To keep up with the rest of the big companies.

c. Corporate universities are a booming industry.

d. Because even small companies need growth.

e. Because a small amount of investment can bring huge profits.

ANSWER: Strategic growth can be brought about by professional development.

Explanation:
Check para 9, last sentence

Why are companies setting up corporate universities?

a. Because there are employees in various domains and the leaders too are experts in each domain.

b. Because there are millions of employees and these universities can earn a huge profit.

c. Because there are challenges of scale and complexity and good governance issues that can be
addressed through corporate training.

d. Because academic education does not help the new hires in their job.

e. Because e-learning is the fastest and most reliable method to teach everything from technology to
culture.
ANSWER: Because there are challenges of scale and complexity and good governance issues that can
be addressed through corporate training.

Explanation:
a. There is no mention anywhere of company leaders being domain experts
b. Profits from universities are not mentioned anywhere
c. Look at paragraphs 5,6 and 7
d. Para 8 does talk about bringing entry level hires at par with company peers. But the passage does
not say that academic education does not help at all.

The passage is in favor of:

I) Bringing entry level hires at par with the company peers


II) Good governance should include- managing reporting relations, finances and technical development.
III) Standardizing employee response to day-to-day challenges.

a. Only I

b. Only II

c. I and III

d. II and III

e. Only III

ANSWER: I and III

Explanation:
I. Check para 8, sentence 2
II. Check para 7, sentence 1 (technical development not mentioned)
III. Check para 9, sentence 2

Which of these is false in the context of the passage?

I. The pilot course offered at Crotonville was much shorter in duration compared to the executive
courses today.
II. Till today GE continues to be the only company to provide proper classroom training to its employees.
III. Culture has to be learned from leaders; e-learning modules will not do.

a. only I

b. II and III

c. I and III
d. only III

e. I and II

ANSWER: I and II

Explanation:
i. Check para 1, sentence 4
ii. Check para 2 & 3
iii. Check the last sentence of para 10

Reading Comprehension
There are no rigid rules to do research. Like cycling or swimming you do it by doing it. As you do it, you
learn to do it better. Or give up. Analytical, experimental and other technical skills will prove useful.
Knowledge of your field of research will save time. On many instances of doubt, common sense is
sufficient and on many occasions of failure, perseverance a must. Extraordinary intelligence is welcome.
Genius is rare.

Obviously, communication in all forms is essential for recognition and success. Research and semi-
technical articles, monographs, formal and popular books, seminars, talks and lectures, incentive-less
web writing, all could and should be done consistently. Staying motivated for enough years is crucial to
enhance your chance for delivering, if not anything outstanding, something substantial. To give a weak
analogy: Just a boundary or a six makes us literally a one hit wonder. Scoring a century, with or without
boundaries or sixers, is still acommendable effort.

Can research method(s) or method(s) to do science be capsuled into an algorithm yielding assured
success, irrespective of the user? Such attempts of algorithms are mostly met with counter examples of
scientific advancements that happened without practicing those algorithms. To reproduce from Cosma’s
Scientific Method note, a quote by Sir Peter Medawar:
If the purpose of scientific methodology is to prescribe or expound a system of inquiry or even a code of
practice for scientific behavior, then scientists seem to be able to get on very well without it. Most
scientists receive no tuition in scientific method, but those who have been instructed perform no better
as scientists than those who have not. Of what other branch of learning can it be said that it gives
its proficients no advantage; that it need not be taught or, if taught, need not be learned?

What is the central theme of the passage?

a. The use of algorithms to reach to the next level of scientific research

b. The rules and techniques that may be used for scientific research.

c. Sir Peter Medawar’s view that methodology for scientific research is useless

d. That intelligence and genius are not essential for research

e. The importance of tenacity and perseverance above any scientific methodology for successful
research.
NSWER: The importance of tenacity and perseverance above any scientific methodology for successful
research.

Explanation:
Option 1 - Is untrue
Option 2 - Rules and techniques have been summed up only in the first para
Option 3 - Sir Peter’s view is a supporting argument and not the main theme
Option 4 - Just one sentence on intelligence and genius
Option 5 - The entire passage talks of all the qualities and skill sets needed for research and that no
set of rules is full proof or mandatory.

What does the author think about user-independent algorithms that promise to ensure correct results?

a. The author thinks they are very important for ensuring quality in scientific studies.

b. The author thinks that these algorithms do not work most of the time.

c. The author thinks that these algorithms are cumbersome and the results are not good enough to
justify the use of these time consuming techniques.

d. The author thinks that many scientific studies have yielded great results even without the use of these
algorithms.

e. The author has no opinion on the use of such complex algorithms.


ANSWER: The author thinks that many scientific studies have yielded great results even without the
use of these algorithms.

Explanation:
See para 3, sentence 1 & 2

In the context of the passage, choose the word which is most opposite in meaning to the given word.

Proficients

a. Incompetent people

b. Experts

c. Amateurs

d. Students

e. Teachers
ANSWER: Incompetent people

Explanation:
The proficients in the field of mathematics have taken to pursuing research work. Those who were
incompetent however, chose a different field of study.

In the context of the passage, choose the word which is most opposite in meaning to the given word.

Commendable

a. Reprehensible

b. Mediocre

c. Laudable

d. Unexpected

e. Common

ANSWER: Reprehensible

Explanation:
Their efforts to open a clubhouse with the little money they had saved were commendable. The
attempt the business owner made to sabotage their work was reprehensible.

Passage

I first got to know about the Arctic circle in my Geography class when I was 12 years old. The …(a)… of a
land where the sun never sets was fascinating. I cannot remember what exactly made me so …(b)….
about this phenomenon, but I can recall myself …(c)… about places where I can witness one. The beauty
of Norway enthralled me in no time and it became number one on my travel bucket list.

When I was starting out to travel around the world for a year in April 2014, I …(d) Scandinavia from the
list. Too expensive, I thought. Two months later, when I was traveling in Poland, I realized it would be a
shame to come so close to Norway and yet not see its many wonders. So I booked the train reservation
and headed for Norway for a few days.

Norway did not …(e)…. The majestic mountains, stunning landscapes, famous fjords, stunning cities and
tiny islands of wonder-there was so much to explore in Norway.

In the above passage there are blanks, each of which has been numbered. These numbers are
printed below the passage and against each five words are suggested.

Choose the one that fills the blank numbered 'e' most appropriately.

a. Disgruntle
b. Disappoint

c. Frustrate

d. Depress

e. Satisfy

ANSWER: Disappoint

Explanation:
Norway was beautiful. So it did not disappoint. Disgruntle, frustrate and depress carry stronger
meanings that do not apply here.

In the above passage there are blanks, each of which has been numbered. These numbers are printed
below the passage and against each five words are suggested.

Choose the one that fills the blank numbered 'd' most appropriately.

a. Criss-crossed

b. Added

c. Crossed

d. Deducted

e. Subtracted

ANSWER: Crossed

Explanation:
Read the following sentence. Scandinavia is too expensive. Hence, she cancelled or ‘crossed’ it from
her list. You cannot ‘deduct’ or ‘subtract’ items from a list.

In the above passage there are blanks, each of which has been numbered. These numbers are printed
below the passage and against each five words are suggested.

Choose the one that fills the blank numbered 'c' most appropriately.

a. Looking up

b. Researching

c. Reviewing

d. Assessing
e. Analysing

ANSWER: Researching

Explanation:
The author is curious about the phenomenon, hence she is researching about places where it can be
witnessed. ‘Looking up’ is not followed by ‘about’. Reviewing, assessing and analyzing are incorrect in
this context. Review, assess and analyze are more appropriate there is a plan to follow them up with
some real action.

In the above passage there are blanks, each of which has been numbered. These numbers are printed
below the passage and against each five words are suggested.

Choose the one that fills the blank numbered 'b' most appropriately.

a. Curious

b. Relieved

c. Keen

d. Perplexed

e. Nosy

ANSWER: Curious

Explanation:
When she came to know about this phenomenon it made her curious. A person is ‘keen on’
something.

In the above passage there are blanks, each of which has been numbered. These numbers are printed
below the passage and against each five words are suggested.

Choose the one that fills the blank numbered 'a' most appropriately.

a. Thought

b. Idea

c. Concept

d. Imagination

ANSWER: Idea

Explanation:
‘The idea of’ is generally followed by a noun or noun phrase. ‘The thought of ‘ is generally followed by
an –ing word. For example: The thought of jumping gave me the shivers. Since the author had never
been there it was not the existence itself, but the idea of such a land existing that the author found
fascinating. ‘Concept’ and ‘imagination’ are inappropriate in this context.

One Word Substitutes

Choose the correct alternative which can be substituted for the below given word/ sentence.

A person who talks in sleep is called as

a. Philatelist

b. Somnambulist

c. Somniloquist

d. Oneirocritic

ANSWER: Somniloquist

Explanation:
A person who talks in sleep is called as Somniloquist
Suffix 'ist' is used to denote a person who is skilled or expert in something.
The word 'Somniloquist' is a Latin word. 'Somni' means sleep and 'loqui' means to talk.

The meanings of remaining words are:

Somnambulist - A person, who walks in sleep


Oneirocritic –A person who interprets dreams
Philatelist – A person who collects stamps

A person who is bad in spellings is called as

a. Calligraphist

b. Cartographer

c. Choreographer

d. Cacographist

ANSWER: Cacographist

Explanation:
The correct option is (d) Cacographist.
A person who is bad in spellings is called as Cacographist.
The meanings of other words are:
1) Calligraphist – A person writing beautifully. (Beautiful handwriting)
2) Cartographer – A person who draws maps. The study and practice of making maps is called as
Cartography.
3) Choreographer – A person who teaches dance

Fear of needles is known as :

a. Ergophobia

b. Nyctophobia

c. Trypanophobia

d. Necrophobia

ANSWER: Trypanophobia

Explanation:
Fear of needles is known as Trypanophobia.
The word 'phobia' means 'fear'.
The meanings of remaining words are:
1) Ergophobia – Fear of work
'Ergon' is a Greek word for 'Work'
2) Nyctophobia - Fear of darkness.
'Nycto' means night. This phobia is characterized by severe fear of the dark
3) Necrophobia - Fear of death or dead things
'Necro' means 'death/dead bodies'. Hence, the fear of death/dead bodies is called as 'Necrophobia'

The person who knows everything

a. Omnipresent

b. Omnipotent

c. Omniscient

d. Oblivious

ANSWER: Omniscient

Explanation:
The meanings of the words are:

Omnipresent - present everywhere


Omnipotent – To have unlimited power
Omniscient - The person who knows everything
Oblivious – Ignorant person (A person who is unaware of everything)
A person involving in an activity for pleasure and not money is called as ___

a. Amateur

b. Follower

c. Altruist

d. Antiquarian

ANSWER: Amateur

Explanation:
The meanings of given words are:
Amateur - A person involving in an activity for pleasure and not money
Follower – believer, supporter, etc.
Altruist - A who works for the good of others.
Antiquarian – Person interested in antiquities (ancient objects)

The life history of a person written by an author is called as ___________

a. Autobiography

b. History

c. Bibliography

d. Biography

ANSWER: Biography

Explanation:
The life history of a person written by an author is called as Biography

The meanings of other words are:

a. Autobiography - The life history of a person written by himself is called as Autobiography. (Eg:
'Wings of Fire' – Dr. A.P. J. Abdul Kalam)
b. History - Study of past events
c. Bibliography - List of all of sources used in research work

A person who eats too much is called as _______

a. Glutton
b. Nibbler

c. Cannibal

d. Omnivore

ANSWER: Glutton

Explanation:
The meanings of other words are:

Glutton - A person who eats too much


Nibbler – A person who eats small and quick bites
Cannibal – One who eats the human flesh
Omnivore – Animal/Person consuming food both plant and animal origin

Place for ammunition and weapons is called as ________

a. Asylum

b. Arsenal

c. Archives

d. Acoustics

ANSWER: Arsenal

Explanation:
Place for ammunition and weapons is called as Arsenal.
The meanings of other words are:
Asylum – Hospital for mad people
Archives – Place of collecting public/government/historical records.
Acoustics – Science of sound

An examination of a dead body is called as

a. Amputate

b. Autopsy

c. Morgue

d. Crusade
ANSWER: Autopsy

Explanation:
The meanings of other words are:

Amputate - To cut off an infected part of a person's body


Autopsy - An examination of a dead body
Morgue - A place where dead bodies of victims or accidents are kept for identification.
Crusade - A religious war.

'A large enclosure for keeping birds' is also called as __________

a. Zoo

b. Aquarium

c. Aviary

d. Homicide

ANSWER: Aviary

Explanation:
The meanings of the words are:

Zoo - A large enclosure for wild animals


Aquarium - A large enclosure for fish
Aviary - A large enclosure for keeping birds
Homicide - Killing of human beings

Ordering of Words

Rearrange the following parts (1, 2, 3 and 4) in proper sequence to obtain a correct sentence.

1. its best
2. is simply
3. science
4. the common sense at

a. 1, 2, 4, 3

b. 4, 2, 1, 3
c. 2, 3, 4, 1

d. 3, 2, 4, 1

ANSWER: 3, 2, 4, 1

Explanation:
The correct option is (d) 3,2,4,1.
Science is simply the common sense at its best.
The order of remaining options (a, b, c) cannot make a meaningful sentence.

In the question below, there is a sentence with jumbled up parts. Rearrange these parts, which are
labelled A, B, C and D to produce the correct sentence. Choose the proper sequence.

a. She saw her opportunity


b. to make amends
c. when he came to her home
d. to borrow some sugar.

a. bcda

b. bcad

c. adbc

d. abcd

ANSWER: abcd

Explanation:
The correct option is abcd because she only gets a chance to make amends when something happens
(i.e he came to borrow sugar). Another possible option could be cdab, but this option is not available
in the set of possible choices.

In the question below, there is a sentence with jumbled up parts. Rearrange these parts, which are
labelled A, B, C and D to produce the correct sentence. Choose the proper sequence.

A. I wanted to tell her


B. not to talk to him
C. not listen to me.
D. but she would

a. abcd

b. abdc
c. adbc

d. bcad

NSWER: abdc

Explanation:
The correct answer is (b) because she would not listen to him, so he could not talk to her. The correct
sequence is adbc.

In the question below, there is a sentence with jumbled up parts. Rearrange these parts, which are
labelled A, B, C and D to produce the correct sentence. Choose the proper sequence.

a. The CEO
b. of making the right decision
c. was not scared
d. despite a loss in profits

Correct Sequence:

a. abcd

b. abdc

c. acbd

d. bcda

ANSWER: acbd

Explanation:
“The CEO” is the subject of the sentence not scared of making the decision is the verb or action part
and despite profits were lessening forms part of the predicate or latter half of the sentence. The
entire sentence therefore reads meaningfully if (c) is chosen.

In the question below, there is a sentence with jumbled up parts. Rearrange these parts, which are
labelled A, B, C and D to produce the correct sentence. Choose the proper sequence.

a. She wondered if
b. work hard for a living
c. he would ever
d. because he was so rich.

Correct Sequence:

a. abcd
b. abdc

c. acbd

d. bcda

ANSWER: acbd

Explanation:
Correct sequence is acbd because he need not work hard for a living as he is rich- this is her thought.

In the question below, there is a sentence with jumbled up parts. Rearrange these parts, which are
labelled A, B, C and D to produce the correct sentence. Choose the proper sequence.

a. She danced with joy


b. that she had topped
c. when she found out
d. the board examination.

Correct Sequence:

a. abcd

b. abdc

c. acbd

d. bcda

ANSWER: acbd

Explanation:
She was dancing with joy at some positive news (i.e topping the board exams). While the former is the
subject of the sentence, the latter is the predicate.

In the question below, there is a sentence with jumbled up parts. Rearrange these parts, which are
labelled A, B, C and D to produce the correct sentence. Choose the proper sequence.

a. She was wishing


b. for her
c. for something
d. that was not good

Correct Sequence:

a. abcd

b. abdc
c. acdb

d. bcda

ANSWER: acdb

Explanation:
The correct sentence: She was wishing for something that was not good for her. She refers to the
subject while not good for her is the predicate and was wishing is the verb joining the two.

In the question below, there is a sentence with jumbled up parts. Rearrange these parts, which are
labelled A, B, C and D to produce the correct sentence. Choose the proper sequence.

a. I was certain
b. the management meeting.
c. be allowed to attend
d. that subordinates would not

Correct Sequence:

a. abcd

b. abdc

c. adcb

d. bcda

ANSWER: adcb

Explanation:
Subordinates forms the subject of the sentence while management meeting is the predicate (latter
part of the sentence). Some action is required to connect the subject to the predicate which in this
case is a verb( would not be allowed to attend). Hence the correct sequence is “adcb”.

In the question below, there is a sentence with jumbled up parts. Rearrange these parts, which are
labelled A, B, C and D to produce the correct sentence. Choose the proper sequence.

a. The managing director


b. in listening to her
c. was not interested
d. explanation about why profits were lessening.

a. abcd
b. abdc

c. acbd

d. bcda

ANSWER: acbd

Explanation:
“The managing director” is the subject of the sentence not interested in listening is the verb or action
part and why profits were lessening forms part of the predicate or latter half of the sentence. The
entire sentence therefore reads meaningfully if (c) is chosen.

In the question below, there is a sentence with jumbled up parts. Rearrange these parts, which are
labelled A, B, C and D to produce the correct sentence. Choose the proper sequence.

A swathe of the white stuff

A) stretching for maybe fifteen metres


B) lay across the track in front of us
C) and the dirt track reappeared
D) before it petered out

a. ABDC

b. BCDA

c. BADC

d. DBAC

ANSWER: BADC

Ordering of Sentences

Rearrange the following parts (1, 2, 3 and 4) in proper sequence to obtain a correct sentence.

1. pending because it wasn’t


2. Too many cases still lie
3. is the convict
4. confirmed that the accused really

a. 2, 1, 4, 3

b. 2, 3, 4, 1
c. 1, 4, 3, 2

d. 4, 2, 1, 3

NSWER: 2, 1, 4, 3

Explanation:
The correct option is (a) 2, 1, 4, 3

Too many cases still lie pending, because it wasn’t confirmed that the accused really is the convict.

Rearrange the following parts (1, 2, 3 and 4) in proper sequence to obtain a correct sentence.

1. our country
2. following greater than some
3. Salman Khan has a fan
4. of the best paid sports person in

a. 1, 3, 4, 2

b. 3, 4, 2, 1

c. 4, 1, 3, 2

d. 3, 2, 4, 1

ANSWER: 3, 2, 4, 1

Explanation:
The correct option is (d) 3, 2, 4, 1
Salman Khan has a fan following greater than some of the best paid sports person in our country.

Rearrange the following parts (1, 2, 3 and 4) in proper sequence to obtain a correct sentence.

1) Karan is one of those boys


2) their willingness for
3) joining the INDIAN ARMY.
4) who have expressed

a. 4, 3, 2, 1

b. 1, 3, 4, 2

c. 1, 4, 2, 3

d. 2, 4, 1, 3
ANSWER: 1, 4, 2, 3

Explanation:
The correct option is (c) 1, 4, 2, 3
Karan is one of those boys who have expressed their willingness for joining the INDIAN ARMY.

Rearrange the following parts (1, 2, 3 and 4) in proper sequence to obtain a correct sentence.

1. held on all India basis


2. appear the SSC examination
3. every year many students from
4. all over the country

a. 1, 2, 4, 3

b. 3, 4, 2, 1

c. 2 , 3, 1, 4

d. 2, 4, 1, 3

ANSWER: 3, 4, 2, 1

Explanation:
The correct option is (b) 3, 4, 2, 1

Every year many students from all over the country appear the SSC examination held on all India
basis.

Rearrange the following parts (1, 2, 3 and 4) in proper sequence to obtain a correct sentence.

1. of their passions
2. is a knowledge
3. the knowledge
4. of mankind

a. 3, 2, 4, 1

b. 3, 4, 2, 1

c. 1, 3, 4, 2

d. 4, 3, 2, 1

ANSWER: 3, 4, 2, 1
Explanation:
The knowledge of mankind is a knowledge of their passions is the correct sequence

Rearrange the following parts (1, 2, 3 and 4) in proper sequence to obtain a correct sentence.

1. be, not what


2. always be
3. whom you want to
4. others want to see

a. 1, 2, 4, 3

b. 3, 4, 2, 1

c. 2, 3, 1, 4

d. 4, 3, 1, 2

ANSWER: 2, 3, 1, 4

Explanation:
The correct option is (c) 2, 3, 1, 4

Correct sequence: Always be whom you want to be, not what others want to see.

Pick the word/phrase closest in meaning to the given word:

Succinct

a. Moderate
b. Concise
c. Lengthy
d. Successful

ANSWER: Concise

Explanation:
Succinct: This means very short and brief. Other words similar in meaning are terse, concise and precise.
Succinct is a word which originated in 1400-1500 AD during the late Middle English period.

For example, The presentation was succinct and to the point because the managing director does not
like wasting words.

The given sentence have been divided into four parts out of which a part may contain grammatical
error. Choose the part which has grammatical error or else choose ‘No error’ as your answer.
They were aware(a)/ that there project (b)/ was not going to meet (c)/ the intended target in time (d).

a. a
b. b
c. c
d. d
e. No error

ANSWER: b

Explanation:
(b) that their project.
Their is a possessive pronoun while there indicates location or place and/or existence. For example: It
was their choice (i.e an option which they have)/ He was there all the time (there means to be present in
a particular location or place).

In the sentence given below a part is underlined. Choose the most suitable option that can replace the
underlined part.

There is less need to prepare a presentation, as the clients will not be coming today.

a. hardly any
b. no
c. very little
d. somewhat

ANSWER: hardly any

Explanation:
Less need = hardly any. No need would imply no requirement while very little has a positive meaning
and the sentence does not. Somewhat is not right because it does not match the usage in the sentence.

There are two sentences. Each sentence has a blank in it. Five options are given below the sentence
pair. Choose the option that fits both the blanks.

1.The currency trading session ______ eventually expire.


2.His boss ______ give him a raise

a. May
b. Will
c. Has
d. Wants
ANSWER: Will

Explanation:
Will is the correct choice. Currency trading session has to expire sooner or later (use of the world
eventually indicates this) therefore may cannot be used. Both may and will can be used for the second
sentence, however each is different in meaning. Will implies positive indication raise is to be given while
may indicates lack of certainty.

Complete the following sentences with the most appropriate words/phrases with reference to
grammar, idioms, proverbs and syntax.

________________ should not throw stones.

a. People in glass houses


b. People in crystal houses
c. People in ice houses
d. None of the above

ANSWER: People in glass houses

Explanation:
People in glass houses should not throw stones. (this is an “idiom” or commonly used phrase in the
English language). Writer Geoffrey Chaucer was the first person to use this phrase in the English
language in his work “Troilus”

In the question below, there is a sentence with jumbled up parts. Rearrange these parts, which are
labelled A, B, C and D to produce the correct sentence. Choose the proper sequence.

a. I was certain
b. the management meeting.
c. be allowed to attend
d. that subordinates would not

Correct Sequence:

a. abcd
b. abdc
c. adcb
d. bcda

ANSWER: adcb

Explanation:
Subordinates forms the subject of the sentence while management meeting is the predicate (latter part
of the sentence). Some action is required to connect the subject to the predicate which in this case is a
verb( would not be allowed to attend). Hence the correct sequence is “adcb”.

Choose the correct alternative which can be substituted for the below given word/ sentence.

A word indicating medicines that have no biological value yet create an impact because of
psychological reasons.

a. Albedo
b. Tornado
c. Placebo
d. Albino

ANSWER: Placebo

Explanation:
Placebo refers to medicines that have no biological properties and are simply given to make the patient
believe he/she is being given medication. Placebos like sweet sugar pills are a substitute for real
medicine. They are effective because they impact the mind rather than the body.

In the following question, an idiom or proverb has been underlined. Choose its correct meaning in the
given context from the alternatives given below.

She was scared to death when she realised she had selected the wrong slide for the company
presentation.

a. She actually died from fear


b. She suffered extreme death-like fear
c. She felt death was frightening
d. None of the above

NSWER: She suffered extreme death-like fear

Explanation:
A frightening experience like death has been used to express how she felt when she chose the wrong
slide for a presentation before her work colleagues.

Pick the world exactly opposite in meaning to the given word:

Industrious
a. Agricultural
b. Manufacturing
c. Lethargic
d. Bored

ANSWER: Lethargic

Explanation:
The correct choice is (c) or lethargic because industrious means someone who works very hard/ puts in a
lot of effort for instance, Mary was lethargic while Jenny was efficient and industrious. Agricultural is the
opposite of industrial, not industrious. Manufacturing is a type of industry while bored means
monotony, which has no relation to industrious.

Choose the pair of words which have a relationship similar to that between the given pair of words:

Stench: Fragrance

a. Sound: Clap
b. Sound: Silence
c. Sound: Chatter
d. Sound: Noise

ANSWER: Sound: Silence

Explanation:
Silence is the opposite of sound while clap, chatter and noise are all forms of sound that can be heard.
Stench is the opposite of fragrance therefore the correct choice is (b).

Fastidious

a. Lethargic
b. Fussy
c. Angry
d. Successful

ANSWER: Fussy

Explanation:
Fastidious means fussy and choosy to the extreme.

For example: The restaurant menu was so varied yet Ashok was a fastidious diner who was never
pleased.
he given sentence have been divided into four parts out of which a part may contain grammatical
error. Choose the part which has grammatical error or else choose ‘No error’ as your answer.

They wear going (a)/ to cancel his appointment (b)/ as the CEO (c)/ because of personal enmity (d).

a. a
b. b
c. c
d. d
e. No error

ANSWER: a

Explanation:
(a) They were going:
Were is different from wear which means to put on. For example, Neeta wears expensive clothes/ You
were going to the office till you fell ill.

n the sentence given below a part is underlined. Choose the most suitable option that can replace the
underlined part.

She is dancing to the music of her boss’s whims.

a. tunes
b. songs
c. whistles
d. sounds

ANSWER: tunes

Explanation:
Dancing to the tunes is a phrase used in English. Dancing to songs, whistles and sounds is possible, but it
does not convey the same meaning.

There are two sentences. Each sentence has a blank in it. Five options are given below the sentence
pair. Choose the option that fits both the blanks.

1. He _____ fill the form


2. I don’t understand why he ______ always go to the office before time.

a. Was
b. Will
c. Has
d. Wants
ANSWER: Will

Explanation:
Will is the correct choice. The form cannot be was, has or wants filled because none of these words is
correct usage. Similarly, he will go (implying a future action) rather than past (has. was) and though
wants can be used, the correct sentence in this case would be “I don’t understand why he always wants
to go to the office before time”.

Complete the following sentences with the most appropriate words/phrases with reference to
grammar, idioms, proverbs and syntax.

A stitch in time_______________

a. Saves till nine


b. Saves none
c. Saves till eight
d. None of the above

NSWER: Saves till nine

Explanation:
A stitch in time saves till nine- meaning a timely stitch can prevent the cloth from tearing for a long
period of time. This phrase is used to explain why it is always better to choose proactive approach.

For example: As he had prepared for the exam well in advance, he discovered why a stitch in time saves
till nine when he got good marks.

In the question below, there is a sentence with jumbled up parts. Rearrange these parts, which are
labelled A, B, C and D to produce the correct sentence. Choose the proper sequence.

a. She was wishing


b. for her
c. for something
d. that was not good

Correct Sequence:

a. abcd
b. abdc
c. acdb
d. bcda

ANSWER: acdb
Explanation:
The correct sentence: She was wishing for something that was not good for her. She refers to the
subject while not good for her is the predicate and was wishing is the verb joining the two.

Choose the correct alternative which can be substituted for the below given word/ sentence.

A word meaning happy and joyful

a. Infuriated
b. Exhilarated
c. Despondent
d. None of the above

ANSWER: Exhilarated

Explanation:
While infuriated means angry, despondent means sad. Exhilarated is the right choice.

In the following question, an idiom or proverb has been underlined. Choose its correct meaning in the
given context from the alternatives given below.

She realised there was no use crying over spilt milk when she saw the interviewer promising the job to
someone else.

a. She has spilt milk


b. She realised it was useless to worry about something which cannot be changed
c. She was worried over the spilt milk
d. None of the above

NSWER: She realised it was useless to worry about something which cannot be changed

Explanation:
Just like milk cannot be put back into the vessel after it has spilled, similarly you cannot go back in time
and change things which have not worked out for you. For example: Cursing yourself for not studying
enough after

Pick the world exactly opposite in meaning to the given word:

Question

a. Response
b. Query
c. Issue
d. Conflict

ANSWER: Response

Explanation:
Response is another word for answer or solution. It is the opposite of question in meaning. For example,
For every question of his, she had an equally strong response.

Choose the pair of words which have a relationship similar to that between the given pair of words:

Pleasant: Dislikable

a. Sorry: Defiant
b. Guilty: Innocent
c. Happy: Overjoyed
d. Sad: Sorrowful

ANSWER: Guilty: Innocent

Explanation:
Pleasant is the opposite of dislikable for example Harish is a pleasant and happy person while Mahesh is
thoroughly dislikable. Similarly, guilty is the direct opposite of innocent and therefore the correct
answer. The other two options have word pairs which mean the same thing.

Fill in the blanks with appropriate preposition

He comes ______ a family which has a passion _______ playing cricket.

a. from, in

b. off, for

c. of, in

d. of, for

ANSWER: of, for

Explanation:
The correct option is (d) of, for
He comes of a family which has a passion for playing cricket.

Select the most effective word from the options given below to fill in the blank and complete the
sentence meaningfully.
A ___________ of advisers.

a. army

b. board

c. crowd

d. council

ANSWER: council

Explanation:
The correct option is (d) council.

'A army of soldiers'


'Board' is generally used as 'Board of Directors'
Crowd - It is generally a large group of people that are gathered or considered together.
Council - It is a group of people who are selected to make decisions, consult, advice or deliberate.

Select the most effective word from the options given below to fill in the blank and complete the
sentence meaningfully. (Collective nouns)

A _____________ of eggs.

a. collection

b. clutch

c. quiver

d. brood

ANSWER: clutch

Explanation:
'Clutch' is the correct collective noun to be used.
collection – The word 'collection' is collective noun used for stamps. (A collection of stamps)
quiver – The word 'quiver' is the collective noun used for arrows. (A quiver of arrows)
brood – Brood is a collective noun used for chickens.

Select the appropriate word which makes the sentence meaningful.

The teacher often _____ her temper

a. loses
b. looses

ANSWER: loses

Explanation:
The correct option is (a) loses.
The meanings of the words are:
Lose is a verb. It means 'unable to find, misplace, suffer etc.'
Loose – It means 'loosened, stretched, etc.'
Example in a sentence:
Loose – My new dress is a bit loose.

Choose the correct alternative which can be substituted for the below given word/ sentence.

Cultivation, management, and study of individual trees is called as __________

a. Arboriculture

b. Sericulture

c. Horticulture

d. Viticulture

ANSWER: Arboriculture

Explanation:
Arboriculture – It is the study of individual tree or shrubs, etc. It includes cultural techniques such as
selection, planting, training, fertilization, pest and pathogen control, pruning, shaping, and removal. A
person studying arboriculture is called as ' arborist' or an 'arboriculturist'
Sericulture –Rearing of silk worms is known as Sericulture.
Horticulture – It is the branch of agriculture that deals with the art, science, technology, and business of
growing plants. Cultivation of medicinal plants, fruits, vegetables, nuts, seeds, herbs, sprouts, etc.
Viticulture – The science, production, and study of grapes is called as Viticulture.

Fill in the blanks with appropriate preposition

Instead of firing ______ the terrorist, the guard fired _______ the mob.

a. on, at

b. at, upon

c. on, on

d. none of these
ANSWER: at, upon

Explanation:
The correct option is (b) at, upon.

Instead of firing at the terrorist, the guard fired upon the mob.

Fill in the blanks with appropriate preposition

She has applied ___ the principal ____ the post of lab assistance.

a. to, to

b. to, at

c. to, for

d. for, at

ANSWER: to, for

Explanation:
She has applied to the principal for the post of lab assistance.
'To' is used to indicate the direction of something towards place, person or thing, here it is to the
principal.
'For' is used to indicate the use of something.

Select the appropriate word which makes the sentence meaningful.

The _________ of this college is very strict.

a. principle

b. principal

ANSWER: principal

Explanation:

The word 'principle' means a fundamental truth or law that has many applications in different fields. It is
a rule or belief about what is right and wrong .

The word 'principal' is a noun. Principal may be the head of school, college, etc.

Example in a sentence :

1) The principal of this college is very strict.


2) It is governed by the basic principle which uses two parts one uses statement of sources and another
uses the use of funds.

Rearrange the following parts (1, 2, 3, 4 and 5) in proper sequence to obtain a correct sentence.

1) influenced life around the world, including


2) developed devices that greatly
3) phonograph, motion picture camera
4) Thomas Alva Edison
5) and the electric light bulb

a. 3, 1, 4, 2, 5

b. 4, 2, 1, 3, 5

c. 4, 1, 3, 5, 2

d. 3, 5, 1, 2, 5

ANSWER: 4, 2, 1, 3, 5

Explanation:
The correct option is (b) 4, 2, 1, 3, 5

Thomas Alva Edison developed devices that greatly influenced life around the world, including
phonograph, motion picture camera and the electric light bulb.

Select the appropriate word which makes the sentence meaningful.

The planets are not ___________, they move round the sun

a. stationary

b. stationery

ANSWER: stationary

Explanation:
The correct option is (a) stationary
The planets are not stationary, they move round the sun.
The word 'stationary' means not moving and 'stationery' means writing papers, pens, envelopes, etc.
(writing and official supplies)
For Example:
1) The office boy is in charge for ordering the stationery.
2) The boy rashly drove the car and hit a stationary bus.
Fill in the blanks with appropriate preposition

The diagram I am referring ____ is given _____ page twenty.

a. to, on

b. to, at

c. at, on

d. at, in

ANSWER: to, at

Explanation:
The correct option is to, at
The diagram I am referring to is given at page twenty.
The preposition 'at' is used to show an exact position or particular place.

Fill in the blanks using the correct form of the word given in the bracket.

Kate's mother had firm ________ (believe) in her.

a. believed

b. believing

c. belief

d. believes

ANSWER: belief

Explanation:
The correct option is (c) belief
Kate's mother had firm belief in her.
Belief is a noun which refers to the act of accepting something as true without any proof. This noun is
used if a person is a talking about faith or trust in someone.

Rearrange the following parts (1, 2, 3 and 4) in proper sequence to obtain a correct sentence.

1. recently launched
2. a fund to
3. the Department of Biotechnology
4. tackle anti-microbial resistance
a. 2, 1, 3, 4

b. 3, 1, 2, 4

c. 1, 3, 4, 2

d. 4, 3, 2, 1

ANSWER: 3, 1, 2, 4

Explanation:
The correct option is (b) 3, 1, 2, 4
The Department of Biotechnology recently launched a fund to tackle anti-microbial resistance
Option (a), (c) and (d) do not form a correct sentence.

Select the most effective word/word-group from the given options to fill in the blank and complete
each sentence meaningfully.

The willingness of John to take up any responsibility is a sign of ___________ .

a. status

b. greed

c. maturity

d. strength

ANSWER: maturity

Explanation:
The willingness of John to take up any responsibility is a sign of maturity.
Options (a), (b), (d) 'status', 'greed', or 'strength' are not so appropriate and make no sense.
The most effective word which completes the sentence meaningfully is 'maturity'.
'Maturity' here means a stage where John is capable of taking new challenges.
The stage of having willingness simply means that the person is matured.

Select the most effective word/word-group from the given options to fill in the blank and complete
each sentence meaningfully.

The secretory assured the shareholders that the management would __________ itself to increase
production.

a. acquit
b. change

c. exert

d. manage

ANSWER: exert

Explanation:
The secretory assured the shareholders that the management would exert itself to increase production.
Option (a) 'acquit itself to' is incorrect
Option (b) 'change itself to' is incorrect. The word 'change itself' would mean admission of great failure.
Option (c) 'exert' means 'to make an effort' – an extra effort. Hence, it makes sense.
Option (d) 'management would manage' is tautology (needless repetition of words)
Hence, is incorrect.

Choose correct pair of words in the same order to fill in the blank spaces and make the sentence
meaningful.

We can ________ breakfast in a 5-star hotel, but we _________ street food.

a. decide, liked

b. afford, prefer

c. plan, enjoyed

d. aspire, commute

ANSWER: afford, prefer

Explanation:
Option a) decide and liked are irrelevant and add no meaning to the sentence.
Option b) 'afford' and 'prefer' are relevant and make the sentence meaningful
Option c) plan and enjoyed are also irrelevant because 'we can plan' says that they still have to plan and
the word 'enjoyed' says that they have already enjoyed. This makes no sense.
Option d) The words aspire and hope do not fit in the given context.

Select the most effective word/word-group from the given options to fill in the blank and complete
each sentence meaningfully.

The boy was about to park his bicycle, when a passing vehicle _________ him down.

a. pushed
b. threw

c. knocked

d. kicked

ANSWER: knocked

Explanation:
The boy was about to park his bicycle, when a passing vehicle knocked him down.

Option (a) 'push' lacks the force of a passing 'vehicle'


Option (b), (d) The word 'threw' and kicked are generally used when it is the act of a person. But here,
the boy fell down due to a passing vehicle. Hence, it is irrelevant.
Option (c) 'knocked' is the most appropriate word. 'Knocked him down' means to hit or push someone
accidentally.

Select correct pair of words in the same order to fill in the blank spaces and make the sentence
meaningful.

The factory workers _________ threatened to launch a strike for next 15 days to _______ their
demands.

a. have, press

b. has, press

c. did, get

d. nearly, fulfill

ANSWER: have, press

Explanation:
The factory workers have threatened to launch a strike for next 15 days to press their demands.
In this sentence,'press' means 'to put emphasis on'
Option (b) cannot be used because the word 'has' is used along with pronouns such as he, she, who, etc.
and the singular nouns.
Option (c) The word 'did' is irrelevant and cannot be used.

Option (d) The workers want the management to fulfill their demands, so the right construction of the
sentence to use this option (nearly,fulfill) has to be

The factory workers nearly threatened to launch a strike for next 15 days to get their demands fulfilled.

Select the most effective word from the options given below to fill in the blank and complete the
sentence meaningfully.
She had hardly seen her mother ______ she stopped crying.

a. then

b. than

c. when

d. so

ANSWER: when

Explanation:
Here, the sentence intends to communicate that, As soon as the girl saw her mother, she stopped
crying.
The action of seeing her mother and stopped crying are performed simultaneously.

Hardly is always followed by the word when.

The confusion here may arise between the options 'b' and 'c' i.e. than and when.

Here, you must always remember that the conjunction used with 'Hardly' is 'when”

Example: Hardly had the husband entered the house when the wife started complaining.

Choose correct pair of words in the same order to fill in the blank spaces and make the sentence
meaningful.

Mothers _________ led her child on the _____ track.

a. bewilderment, appropriate

b. predicament, proper

c. ignorance, wrong

d. agony, funny

ANSWER: ignorance, wrong

Explanation:
Mothers ignorance led her child on the wrong track.
In option (a) – The word 'bewilderment' means 'confused state of mind' and appropriate means
'correct'. Hence, it is irrelevant in the given context.
In option (b) – The word 'predicament' means dangerous situation and 'proper' means the right thing.
Hence, the words 'predicament and proper' cannot be used in the blank spaces, because it makes no
sense.
In option (c) – The word ignorance and wrong, add proper meaning to the sentence and make it
meaningful.
Option (d) Agony means hurt which doesn't go well with funny. Hence it is irrelevant.

Choose one word which can meaningfully replace the underlined words in both the sentences without
changing the meaning of original sentences.

1. Teachers understanding of the English subject was really good.


2. The mother had to hold on to the door-handle with all her strength to get the baby out.

a. pull

b. grasp

c. seize

d. compass

ANSWER: grasp

Explanation:
Here, 'understanding' is a noun and 'hold' a verb.
Of these 'grasp' as a noun means 'understanding' and as a verb means 'hold' or 'hold on to'. Hence, it
can replace 'understanding' in (1) and 'hold on to' in (2).
Pull, seize and compass are irrelevant and do not complete both the sentences meaningfully.
Therefore, the sentences can be written as:
(I) Teachers grasp of the English subject was really good.
(II) The mother had to grasp the door-handle with all her strength to get the baby out.

Select the most effective word from the options given below to fill in the blank and complete the
sentence meaningfully.

The present __________ green forest of the area is a far cry from the dirty, foul smelling landfill that it
was a decade ago.

a. blue

b. complete

c. beautiful

d. quiet

e. lush
ANSWER: lush

Explanation:
The present lush green forest of the area is a far cry from the dirty, foul smelling landfill that it was a
decade ago.
The options blue, complete and quite are irrelevant and do not make the sentence meaningful.

The confusion can be between the words beautiful & lush.

'Beautiful' does make the sentence meaningful but the contrast of the green forest of the area in the
sentence is a dirty, foul smelling landfill that demands a more appropriate and enhancing adjective.

'lush' when used of nature, plants, trees, gardens, forests, etc. means 'rich', 'verdant' (green). Hence,
lush is the appropriate word to be used for a green forest, grass, garden etc.

There are two sentences. Each sentence has a blank in it. Five options are given below the sentence
pair. Choose the option that fits both the blanks.

1. Anisha decided to resign after she was passed ___ for promotion again.
2. I will go __ to the shop if you will come with me.

a. Under

b. In

c. Over

d. About

ANSWER: Over

Explanation:
The answer is (b) because sentence 1 requires the use of the idiom passed over. You cannot go under, or
about a shop though you can go in to it. But the correct word for both sentences is over in this context.
(c) is the right answer.

There are two sentences. Each sentence has a blank in it. Five options are given below the sentence
pair. Choose the option that fits both the blanks.

1. Her husband died 10 years ago; she has been living as a _______ since then.
2. The unhappy _______ was left with no option but to sell her jewellery to support her family, after
he died.

a. widower
b. widow

c. celibate

d. none of the above

ANSWER: widow

Explanation:
The correct term for a person whose husband is dead is widow. This makes it the right choice for
sentence 1. Sentence 2 is also about a male figure dying and the only options that are for female are
widow and celibate. But widow is the correct term among the two choices. The right answer is (b).

There are two sentences. Each sentence has a blank in it. Five options are given below the sentence
pair. Choose the option that fits both the blanks.

1. There is no joy __ failure.


2. He is ___ his shop today.

a. Within

b. In

c. Inn

d. About

ANSWER: In

Explanation:
In is correct because though joy cannot be within or about failure and , he cannot be within a shop. Inn
and about are both wrong usage in the two sentences. Therefore, (b) or in is the correct choice.

There are two sentences. Each sentence has a blank in it. Five options are given below the sentence
pair. Choose the option that fits both the blanks.

1. He went ___ in a huff.


2. She was ____ to Mumbai for a holiday

a. Of

b. Are

c. If

d. Off
ANSWER: Off

Explanation:
Of means belonging to something while off means to move on. Both sentences imply movement of
some type and if as well as are are clearly wrong. The right answer is therefore (d)

There are two sentences. Each sentence has a blank in it. Five options are given below the sentence
pair. Choose the option that fits both the blanks.

1. He had pinned his hopes ___ false promises.


2. She went __ to win the competition.

a. At

b. On

c. Under

d. After

ANSWER: On

Explanation:
Hopes have to be pinned on something, rather than at, under or after. Similarly, winning the
competition is something she went on to do (Past Continuous Tense). Hence choice (b) is right

There are two sentences. Each sentence has a blank in it. Five options are given below the sentence
pair. Choose the option that fits both the blanks.

1. He _____ fill the form


2. I don’t understand why he ______ always go to the office before time.

a. Was

b. Will

c. Has

d. Wants

ANSWER: Will

Explanation:
Will is the correct choice. The form cannot be was, has or wants filled because none of these words is
correct usage. Similarly, he will go (implying a future action) rather than past (has. was) and though
wants can be used, the correct sentence in this case would be “I don’t understand why he always wants
to go to the office before time”.

There are two sentences. Each sentence has a blank in it. Five options are given below the sentence
pair. Choose the option that fits both the blanks.

1.The currency trading session ______ eventually expire.


2.His boss ______ give him a raise

a. May

b. Will

c. Has

d. Wants

ANSWER: Will

Explanation:
Will is the correct choice. Currency trading session has to expire sooner or later (use of the world
eventually indicates this) therefore may cannot be used. Both may and will can be used for the second
sentence, however each is different in meaning. Will implies positive indication raise is to be given while
may indicates lack of certainty.

There are two sentences. Each sentence has a blank in it. Five options are given below the sentence
pair. Choose the option that fits both the blanks.

1. There is no hope __ her heart.


2. He is ___ his office today.

a. Within

b. In

c. Beyond

d. About

ANSWER: In

Explanation:
In is correct because though hope was be within the heart, he cannot be within an office. Beyond and
about are both wrong usage in the two sentences. Therefore, (b) or in is the correct choice.
There are two sentences. Each sentence has a blank in it. Five options are given below the sentence
pair. Choose the option that fits both the blanks.

…. he know that there has been a theft at his place?

The little boy stood looking at the ..…..grazing in the distance.

a. Did

b. Does

c. Cows

d. Would

e. Horses

ANSWER: Does

Explanation:
‘Does’ is the third person singular of the verb ‘do’. ‘Does’ is also the plural of doe, a female deer.

There are two sentences. Each sentence has a blank in it. Five options are given below the sentence
pair. Choose the option that fits both the blanks.

Never …… your friends when they need you the most.

They spent a week in the …… without any food or water.

a. Belittle

b. Dump

c. Forest

d. Desert

e. House

ANSWER: Desert

Explanation:
A desert(n) is a dry arid full of sand. To desert (v) someone is to leave him unjustly, without any hope of
return.

There are two sentences. Each sentence has a blank in it. Five options are given below the sentence
pair. Choose the option that fits both the blanks.

A …. has been buzzing near my ears all afternoon.

Kites ……. very high in the sky.

a. Fly

b. Mosquito

c. Soar

d. Insect

e. Rise

ANSWER: Fly

Explanation:
A fly (n) is a small flying insect. To fly(v) means to move through air usually using wings.

There are two sentences. Each sentence has a blank in it. Five options are given below the sentence
pair. Choose the option that fits both the blanks.

Maggi has been withdrawn from markets on account of rumors of ..….poisoning.

Hormonal imbalance can often ….. to depression.

a. Children

b. Food

c. Lead

d. Take

e. Direct

ANSWER: Lead

Explanation:
Lead (n) is a chemical element that when consumed can be poisonous. Lead (v) means to guide in the
direction of something.

There are two sentences. Each sentence has a blank in it. Five options are given below the sentence
pair. Choose the option that fits both the blanks.
She was an elegant ……. dancer.

He has ambitious plans of making a trip to the North …….

a. Classical

b. Pole

c. Zone

d. Hemisphere

e. Ballet

ANSWER: Pole

Explanation:
Pole dancing is a form of dance where dancers dance centered around a pole.

The North Pole is the extreme northern end of the earth.

In the following question there are two sentences. Each sentence has a blank in it. Five options are
given below the sentence pair. Choose the option that fits both the blanks.

She was so upset when she was unable to complete her assignment that she threw the .................
against the wall.
He always proclaims that he is as poor as a church ..................

a. Pen

b. Priest

c. Clerk

d. Mouse

e. Book

ANSWER: Mouse

Explanation:
A mouse is a computer hardware. A mouse is also a little four legged pest that often stays in houses and
old buildings. ‘As poor as a church mouse’ is an idiom meaning ‘very poor’.

In the following question there are two sentences. Each sentence has a blank in it. Five options are
given below the sentence pair. Choose the option that fits both the blanks.
Ravi wrote a .....poem.

He jumped the red light and ended up paying a ..............

a. Fine

b. Beautiful

c. Penalty

d. Lump sum

e. Boring

ANSWER: Fine

Explanation:
Fine means nice and beautiful. Fine also means penalty.

In the following question there are two sentences. Each sentence has a blank in it. Five options are
given below the sentence pair. Choose the option that fits both the blanks.

He told the shopkeeper to settle on a ....... price.

We went to the ...... where my daughter bought several hair clips.

a. Party

b. Agreeable

c. Fair

d. Middle

e. Shop

ANSWER: Fair

Explanation:
Fair means just and honest. Fair also means a gathering of people at a specified time and place where
vendors, performers and customers all gather together.

In the following question there are two sentences. Each sentence has a blank in it. Five options are
given below the sentence pair. Choose the option that fits both the blanks.
When he refused to attend the after-dinner party she told him to ....himself.

He bought a designer ....... for himself.

a. Leave by

b. Suit

c. Dress

d. Coat

e. Enjoy

ANSWER: Suit

Explanation:
She asked him to suit himself, i.e., have it his way, or do as he pleases. Suit is also an article of clothing.
‘Dress’ may seem like a probable answer but since he is not going to the party, her asking him to get
dressed does not make sense.

In the following question there are two sentences. Each sentence has a blank in it. Five options are
given below the sentence pair. Choose the option that fits both the blanks.

When the judge entered the bailiff ....... to greet him.

A ....... by any other name smells just as sweet.

a. Stood up

b. Rose

c. Damsel

d. Forgot

e. Pumpkin pie

ANSWER: Rose

Explanation:
Rose is the past tense of rise, which means to get up. Rose is also the name of a flower.

In the question below some part of the sentence has been left blank. Select the option, which
completes the sentence properly taking care of syntax, semantics and meaning.

I did not like what I wore but I did not much care, for I was always of the opinion that …………….

a. shabby clothes is beautiful


b. real beauty was what lay on the inside

c. I will buy new clothes tomorrow

d. I was too happy to notice

ANSWER: real beauty was what lay on the inside

In the question below some part of the sentence has been left blank. Select the option, which
completes the sentence properly taking care of syntax, semantics and meaning.

The dictionary defines patience as the capacity to accept or tolerate delay, trouble or
suffering……………

a. without feeling anger or upset

b. and behave nicely

c. without getting angry or upset

d. by a smile

ANSWER: without getting angry or upset

select the appropriate word which makes the sentence meaningful.

Shahid Afridi is an __________ player.

a. eminent

b. imminent

ANSWER: eminent

Explanation:
The correct option is (a) eminent

'Eminent' means a person who is successful, well-known, outstanding, etc.


'Imminent' means about to happen or likely to occur, possible, etc.
Use of Imminent in a sentence:
Imminent - The earthquake is imminent, so be prepared to face it.

Out of 4 given sentences, select the sentences which are grammatically correct.

1) She knows her mistakes.


2) The teacher asked Kate that why she was late to school.
3) The higher we go, the cooler it is.
4) She had been learning dance for two months.

a. 1 and 2

b. 1, 2 and 3

c. 1, 3 and 4

d. 2 and 3

e. All of the above

ANSWER: 1, 3 and 4

Explanation:
Sentences (1), (3) and (4) are grammatically correct.
Sentence (2) The teacher asked Kate that why she was late to school is incorrect.
Reported speech (Indirect speech) is usually used to talk about the past. Hence, the tense of words is
changed.
Here, in this sentence, 'that' is used before 'why'. In reported speech, 'that' cannot be used before the
words like 'who, whose, whom, when, etc.'

Rearrange the following parts (1, 2, 3 and 4) in proper sequence to obtain a correct sentence.

1. and he who has hope,


2. has hope
3. has everything
4. the person who has health

a. 1, 4, 3, 2

b. 3, 2, 4, 1

c. 3, 4, 1, 2

d. 4, 2, 1, 3

ANSWER: 4, 2, 1, 3

Explanation:
The correct option is (d) 4, 2, 1, 3
The person who has health has hope, and he who has hope has everything.
The order of remaining options (a, b, c) cannot make a meaningful sentence.

Following is a set of four sentences. Choose the sentence which is most appropriate – grammatically,
semantically & logically.
a. Sorry to keep you. How long are you waiting?

b. My apologies for delaying. How long have you been waiting.

c. Sorry to keep you. How long have you been waiting?

d. Sorry for the delay. Since how long are you waiting?

ANSWER: My apologies for delaying. How long have you been waiting.

Following is a set of four sentences. Choose the sentence which is most appropriate – grammatically,
semantically and logically .

a. Hi Harish. Long time no see. How was it going?

b. Hi Harish. We haven’t meet in a long time. How is it going?

c. Hi Harish. We haven’t met in a long time. How is it going?

d. Hi Harish. Long time no meet. How has it been going?

ANSWER: Hi Harish. We haven’t met in a long time. How is it going?

Following is a set of four sentences. Choose the sentence which is most appropriate - grammatically,
semantically and logically.

a. This site will give atleast several no-follow links to your blogs and site from your articles.

b. This site allows you to give several no-follow links to your blogs and site from your articles.

c. This site allow you to give several no-follow links to your blogs and site form your articles.

d. You can get several no-follow link to your blog and site from this article.

ANSWER: This site allows you to give several no-follow links to your blogs and site from your articles.

Following is a set of four sentences. Choose the sentence which is most appropriate – grammatically,
semantically and logically.

a. The word 'discipline' means a training that produces obedience or self-controlled behaviour to the
orders of a proper authority.

b. The word 'discipline' means a training that produces obedience or self-controlled behaviour at the
orders for a proper authority.
c. Discipline’ is a word that produces obedience or self-controlled behavior to the orders of a proper
authority.

d. Discipline is nothing but training that produces obedience and self-control to orders of authority.

ANSWER: The word 'discipline' means a training that produces obedience or self-controlled behaviour
to the orders of a proper authority.

Following is a set of four sentences. Choose the sentence which is most appropriate , grammatically,
semantically & logically.

a. It is highly desirable that you complete the syllabus before the commencement of the annual exams.

b. It is highly desirable for you to complete your syllabus before the commencement of the annual
exams.

c. It is highly desirable if you complete the syllabus before the commencement of the annual exams.

d. It is highly desired if you should complete the syllabus before the commencement of the half yearly
exams.

ANSWER: It is highly desirable that you complete the syllabus before the commencement of the
annual exams.

Following is a set of four sentences. Choose the sentence which is most appropriate – grammatically,
semantically and logically.

a. Young people frequently fall into the trap of assuming that the difficulties they face today are greater
and more troublesome than those faced by previous generations.

b. Young people frequently fall into the trap of assuming that the difficulties they face today are greater
and troublesome than those faced by previous generations.

c. Young people today are trapped by assuming that their difficulties are troublesome and more great
than the previous generations.

d. By assuming they face troublesome difficulties than the previous generation, young people today are
frequently trapped.

ANSWER: Young people frequently fall into the trap of assuming that the difficulties they face today
are greater and more troublesome than those faced by previous generations.

The given sentence has been divided into 4 parts out of which a part may contain grammatical error.
Select the part having grammatical error as answer else mark option 'e' i.e. (No correction required)'
as the answer.
By this time (1) / next year (2) / I had completed (3) /my post graduation. (4)

a. 1

b. 2

c. 3

d. 4

e. No correction required

ANSWER: 3

Explanation:
I had completed is the mistake. The correct option is (c)
The person here talks about completing his post graduation in the future.
Hence, here future perfect tense 'I shall have completed' should be used.
Correct Sentence: By this time next year I shall have completed my post graduation.

The given sentence has been divided into 4 parts out of which a part may contain grammatical error.
Select the part having grammatical error as answer else mark option 'e' i.e. (No correction required)'
as the answer.

I found that dress (1) /more better than(2) / any other dress (3) / in the case.(4)

a. 1

b. 2

c. 3

d. 4

e. No correction required

ANSWER: 2

Explanation:
more better than is the mistake.
The word 'better' itself means 'more good'. 'Much better' can be used, but 'more better' is a slang
usage.

3 forms of adjectives on a scale of comparison are:


Adjective Comparative Superlative

Good Better (more good) Best (most good)

Comparative adjectives compare two things.


Superlative adjectives compare more than two things.

Read each sentence to find out whether there is any grammatical error in it. The error, if any, will be
in one part of the stench. The alphabet of that part is given at the end of each phrase as the answer. If
there is no error, answer is (e)

As they planned a holiday (a)/ to Kashmir (b)/ they were happy that (c)/ it are a beautiful place(d) No
error (e)

a. a

b. b

c. c

d. d

e. No error

ANSWER: d

Explanation:
Certain nouns always take singular verb such as scenery, news, poetry and so on. Kashmir is a place and
the pronoun is singular therefore (is) is correct, not are.
Correct usage: is a beautiful place

The given sentence has been divided into 4 parts out of which a part may contain grammatical error.
Select the part having grammatical error as answer else mark option 'e' i.e. (No correction required)'
as the answer.

The doctors would not (1) / allow anyone to (2) / enter into the (3) / patient's room. (4)

a. 1

b. 2

c. 3

d. 4

e. No correction required
ANSWER: 3

Explanation:
enter into the is the mistake.
'The doctors would not allow anyone to enter the patient's room' is the correct form of the sentence.
The word 'into' cannot be used here, because the word 'enter' means go or come into, which means the
same.

The given sentence has been divided into 3 parts out of which a part may contain grammatical error.
Select the part having grammatical error as answer else mark option 'd' i.e. (No correction required)'
as the answer.

She is my cousin sister(1) / who works as (2) / an Engineer (3)

a. 1

b. 2

c. 3

d. No correction required

ANSWER: 1

Explanation:
She is my cousin sister is the mistake.
Correct form of sentence: She is my cousin who works as an Engineer.
Cousin sister is a slang usage. The word 'cousin' is a gender-neutral noun.
'She is my cousin sister' itself tells that one is talking about a female cousin. Hence, there is no need to
add the word brother/sister after having said the cousin.

The given sentence has been divided into 4 parts out of which a part may contain grammatical error.
Select the part having grammatical error as answer else mark option 'e' i.e. (No correction required)'
as the answer.

The property (1) / of the grandfather (2) / should be divided (3) / equally between his sons. (4)

a. 1

b. 2

c. 3

d. 4

e. No correction required

ANSWER: 4
Explanation:
equally between his sons is the mistake
'The property of the grandfather should be divided equally among his sons' is the correct form of the
sentence.
The word between is always used if the persons or objects are only two. But in this condition, the
number of sons is not specified. Hence, the word 'among' should be used.

The given sentence has been divided into 4 parts out of which a part may contain grammatical error.
Select the part having grammatical error as answer else mark option 'e' i.e. (No correction required)'
as the answer.

He went(1) / to the shop (2) / to buy (3) /a scissor (4)

a. 1

b. 2

c. 3

d. 4

e. No correction required

ANSWER: 4

Explanation:
a scissor is the mistake in the given sentence.
Correct form of sentence: He went to the shop to buy a pair of scissors.
The word scissor is never used in singular form. It is always used in plural form with a pair attached to it.

The given sentence has been divided into 4 parts out of which a part may contain grammatical error.
Select the part having grammatical error as answer else mark option 'e' i.e. (No correction required)'
as the answer.

Every child must make (1) / a resolution of giving up atleast (2) / one of the bad habit (3) / such as
getting up late, biting nails, etc.(4)

a. 1

b. 2

c. 3

d. 4

e. No correction required
NSWER: 3

Explanation:
one of the bad habit is the mistake

Correct form: Every child must make a resolution of giving up at least one of the bad habits such as
getting up late, biting nails, etc.
Here ' atleast one of the bad habit' implies one of many bad habits.
Hence, 'habit' must be in the plural form.

The given sentence has been divided into 3 parts out of which a part may contain grammatical error.
Select the part having grammatical error as answer else mark option 'd' i.e. (No correction required)'
as the answer.

The sceneries (1)/of the Himalayas is (2)/extremely beautiful and excellent.(3)/No correction required
(4)

a. 1

b. 2

c. 3

d. No correction required

ANSWER: 1

Explanation:
The scenery of the Himalayas is extremely beautiful and excellent is the correct form of the given
sentence.
Nouns such as scenery, poetry, etc. are generally used in singular form

In the sentence given below, replace the phrase underlined to make the sentence grammatically
correct. If the sentence is correct as it is, mark (e) i.e. “No correction required” as the answer.

Although she (1)/ is handicapped (2)/ but she (3)/ can run fast.(4)

a. 1

b. 2

c. 3

d. 4

e. No correction required
ANSWER: 3

Explanation:
In this sentence, 'but she' is the mistake.
Correct form: Although she is handicapped yet she can run fast.
Two contrary qualities of a subject can be expressed using the conjunction 'Although ----- yet'
The given statement can also be written as :

1) Although she is handicapped, she can run fast.


2) Though she is handicapped, she can run fast.

Here, the word 'yet' can be replaced by simply using a comma after the word handicapped.
The word 'although' can also be replaced by 'though'

But always remember, 'Although' is always followed by the word 'yet' and not by the word 'but'.

The given sentence has been divided into 5 parts out of which a part may contain grammatical error.
Select the part having grammatical error as answer else mark option 'e' i.e. (No correction required)'
as the answer.

A boy who is (1) / constantly hesitating about (2) / which of any two tasks (3) / he will do first, will
ultimately do either. (4) / No correction required. (5)

a. 1

b. 2

c. 3

d. 4

e. No correction required

ANSWER: 4

Explanation:
he will do first, will ultimately do either is the mistake.

The correct form of sentence should be:


1) 'A boy who is constantly hesitating about which of any two tasks he will do first, will ultimately do
neither. OR
2) A boy who is constantly hesitating about which of any two tasks he will do first, will ultimately not do
either.
The word 'Either' means 'each of two' or 'both' and 'Neither' means 'not one or the other'
The given sentence has been divided into 4 parts out of which a part may contain grammatical error.
Select the part having grammatical error as answer else mark option 'e' i.e. (No correction required)'
as the answer.

The Bill was (1) / pushed through National Assembly (2) / in spite of opposition (3) / but for little
modification. (4).

a. 1

b. 2

c. 3

d. 4

e. No correction required

ANSWER: 4

Explanation:
'The Bill was pushed through National Assembly inspite of opposition but with some modification.' or
'opposition and with little modification'.
The latter means 'with almost no modification'.
Little means no, a little means some, somewhat etc.
For e. g.
“The boy had little money to buy lunch” means “The boy had no money to buy lunch”
“The boy had a little money to buy lunch” means “The boy has some money to buy lunch”

The given sentence have been divided into parts out of which a part may contain grammatical error.
Choose the part which has grammatical error or else choose ‘No error’ as your answer.

Either the manager(a)/ or his subordinates (b)/ had failed in his (c)/ duties, because the project fell
apart (d).

a. a

b. b

c. c

d. d

e. No error

ANSWER: c
Explanation:
(c) is the correct choice because when a singular (manager) and plural (his subordinates) noun are linked
by or or nor, plural pronoun has to be used.
Incorrect: Either the manager or his subordinates had failed in his duties because the project fell apart.
Correct: Either the manager or his subordinates had failed in their duties, because the project fell apart.

Their is used, not his, because it could be either the manager (singular) or his subordinates (plural). His
would imply there is no doubt that it is the manager’s fault only.

The given sentence have been divided into parts out of which a part may contain grammatical error.
Choose the part which has grammatical error or else choose ‘No error’ as your answer.
Which photograph (a)/ was lying there(b)/ before Radha came (c)/ into the room? (d).

a. a

b. b

c. c

d. d

e. No error

ANSWER: a

Explanation:
Whose is used for living persons and which is used for inanimate objects. Obviously the photo belongs
to/is of a person and not a lifeless object. Therefore, which should not be used here.
Incorrect Sentence: Which photograph was lying there before Radha came into the room?
Correct Sentence: Whose photograph was lying there before Radha came into the room?

The given sentence have been divided into parts out of which a part may contain grammatical error.
Choose the part which has grammatical error or else choose ‘No error’ as your answer.

It is improbable (a)/ she would bee (b)/ taken away (c)/ against her will (d).

a. a

b. b

c. c

d. d

e. No error

ANSWER: b
Explanation:
(b) for the presentation
Be is the correct preposition in this sentence. Bee means the insect. The correct sentence is therefore: It
is improbable she would be taken away against her will.

The given sentence have been divided into parts out of which a part may contain grammatical error.
Choose the part which has grammatical error or else choose ‘No error’ as your answer.

There was a new project (a)/ very single day (b)/ for the company trainees (c)/ at the office
headquarters (d).

a. a

b. b

c. c

d. d

e. No error

ANSWER: b

Explanation:
(b) Is: very single day
Should be: every single day

E is missing from every.


Very means to the extreme while every means each. Very is not the correct usage here therefore.

The given sentence have been divided into four parts out of which a part may contain grammatical
error. Choose the part which has grammatical error or else choose ‘No error’ as your answer.

They wear going (a)/ to cancel his appointment (b)/ as the CEO (c)/ because of personal enmity (d).

a. a

b. b

c. c

d. d

e. No error

ANSWER: a
Explanation:
(a) They were going:
Were is different from wear which means to put on. For example, Neeta wears expensive clothes/ You
were going to the office till you fell ill.

The given sentence have been divided into four parts out of which a part may contain grammatical
error. Choose the part which has grammatical error or else choose ‘No error’ as your answer.

They were aware(a)/ that there project (b)/ was not going to meet (c)/ the intended target in time (d).

a. a

b. b

c. c

d. d

e. No error

ANSWER: b

Explanation:
(b) that their project.
Their is a possessive pronoun while there indicates location or place and/or existence. For example: It
was their choice (i.e an option which they have)/ He was there all the time (there means to be present in
a particular location or place).

The given sentence have been divided into four parts out of which a part may contain grammatical
error. Choose the part which has grammatical error or else choose ‘No error’ as your answer.

There was no reason (a) / four the presentation (b)/ to be delayed (c)/ once the conference hall was
empty (d).

a. a

b. b

c. c

d. d

e. No error

ANSWER: b

Explanation:
(b) for the presentation
For is the correct preposition in this sentence. Four means the number, while for is used as a connecting
word or preposition and on rare occasions as a conjunction as well. In this sentence, the use of four is
incorrect as there is no numerical value involved.

The given sentence have been divided into parts out of which a part may contain grammatical error.
Choose the part which has grammatical error or else choose ‘No error’ as your answer.

Much is the amount of pressure (a)/ in the top ranking engineering colleges (b)/ that many students
(c)/ drop out mid-way (d)/ No error (e)

a. a

b. b

c. c

d. d

e. No error

ANSWER: a

Explanation:
‘So much is the ..’

he given sentence have been divided into parts out of which a part may contain grammatical error.
Choose the part which has grammatical error or else choose ‘No error’ as your answer.

Unlawful universities for professional courses(a)/ are today a boom industry (b)/ fuelled by the
ambition of parents (c)/ who wish to see their children become a doctor or an engineer at any cost (d).
No error (e)

a. a

b. b

c. c

d. d

e. No error

ANSWER: b
Explanation:
Booming

The given sentence has been divided into parts out of which a part may contain grammatical error.
Choose the part which has grammatical error or else choose ‘No error’ as your answer.

The whole idea (a)/ of holding a candle light march (b)/ seemed too simple at that point (c)/ that we
laughed it off (d).

a. a

b. b

c. c

d. d

e. No error

ANSWER: c

Explanation:
Seemed so simple that we laughed it off

The given sentence has been divided into parts out of which a part may contain grammatical error.
Choose the part which has grammatical error or else choose ‘No error’ as your answer.

He gave me (a)/ an account of all the people (b) / he met when he (c)/ is on official tour(d).

a. a

b. b

c. c

d. d

e. No error

ANSWER: d

Explanation:
The sentence is in past tense. Only this part is in present tense. ‘when he was on official tour’

In the following questions, a sentence is broken up into parts. One of the parts may have an error. You
are required to identify the part containing the error. If there is no error, choose “none of the above”
as your answer.
What made you/ to go to that/ disreputable place?

a. What made you

b. to go to that

c. disreputable place

d. no error

ANSWER: to go to that

n the following questions, a sentence is broken up into parts. One of the parts may have an error. You
are required to identify the part containing the error. If there is no error, choose “none of the above”
as your answer.

Each of them /have been given /a bouquet and a cake.

a. Each of them

b. have been given

c. a bouquet and a cake

d. no error

ANSWER: have been given

In the following questions, a sentence is broken up into parts. One of the parts may have an error. You
are required to identify the part containing the error. If there is no error, choose “none of the above”
as your answer.

It is time you visit your barber.

a. It was time

b. you visit

c. your barber

d. no error

ANSWER: you visit

In the following questions, a sentence is broken up into parts. One of the parts may have an error. You
are required to identify the part containing the error. If there is no error, choose “none of the above”
as your answer.

If you would have called /as promised /I would have blocked a seat for you.

a. If you would have called

b. as promised

c. I would have blocked a seat for you

d. no error

ANSWER: as promised

In the following questions, a sentence is broken up into parts. One of the parts may have an error. You
are required to identify the part containing the error. If there is no error, choose “none of the above”
as your answer.

There was a lot of discussion about if we should buy the car.

a. There was a lot of

b. discussion about if

c. we should buy the car

d. no error

ANSWER: discussion about if

In the following questions, a sentence is broken up into parts. One of the parts may have an error. You
are required to identify the part containing the error. If there is no error, choose “none of the above”
as your answer.

I have been working in this organization /since last six years and/ I am yet to find any dishonest
person.

a. I have been working in this organization

b. since last six years and

c. I am yet to find any dishonest person

d. no error
ANSWER: since last six years and

The sentence below is divided into three parts. Identify where the error lies. If there is no error, select
the option ‘no error’.

The teacher is angry on the student/ for not submitting the assignment/ on time.

a. The teacher is angry on the student

b. for not submitting the assignment

c. on time

d. no error

ANSWER: The teacher is angry on the student

Verbal Analogies

Choose the pair of words which have a similar relationship to that between the given pair of words:

If Charles Darwin is to biologist, then Archimedes is to ______________.

a. mathematics

b. sociology

c. literature

d. religion

ANSWER: mathematics

Explanation:
The correct option is (a) mathematics

Charles Darwin - He was a biologist and scientist, known for his contributions to evolutionary theory.
Archimedes - He was an ancient Greek mathematician, physicist, engineer, inventor and astronomer.

Choose the pair of words which have a similar relationship to that between the given pair of words:

MERGE : DISCONNECT

a. Gratify : Delight
b. Worship : Devotion

c. Robust : Incapable

d. Intense : Acute

ANSWER: Robust : Incapable

Explanation:
The correct option is (c) Robust : Incapable
'Disconnect' is the antonym of the word 'Merge'. Hence, the correct option is (c) which has similar
relationship between the words.
Robust means 'powerful', 'prosperous', 'tough', etc.
Option (a), (b) and (d) have similar relationship between the given pair of words. These words are
synonyms.

Choose the pair of words which have a similar relationship to that between the given pair of words:

If climatology is to climate, then study of birds ___________

a. Apiology

b. Ornithology

c. Entomology

d. Cetology

ANSWER: Ornithology

Explanation:
The word 'Ology' means 'study of' something.
Apiology – Study of bees
Ornithology – Study of birds
Entomology – Study of insects. The word 'Entomon' means insects and hence its study is called as
'Entomology'
Cetology – Study of whales, dolphins and porpoises. Cetelogists are those who practice cetelogy. They
understand and explain evolution, distribution, morphology, etc.

Choose the pair of words which have a similar relationship to that between the given pair of words:

Equilibrium : balance

a. Acquit : blame

b. Bold : cautious

c. Elevation : height
d. Endanger : guard

ANSWER: Elevation : height

Explanation:
Equilibrium and Balance are synonyms. Hence, the correct option having a similar relationship to that
between the given pair of words is Elevation : height

Option (a) : The meaning of acquit is let go, etc.. Blame is antonym of the word 'Acquit'.
Option (b) : The meaning of bold is fearless, daring, etc.. Cautious means to be careful, which is antonym
of the word 'bold'.
Option (d) : The meaning of Endanger is hazard, threaten, etc. Guard means defender, which is antonym
of the word 'Endanger'.

Choose the pair of words which have a similar relationship to that between the given pair of words:

Aligator : reptiles

a. Monkey : amphibians

b. Monkey : mammals

c. Monkey : aquatics

d. Monkey : rhodents

ANSWER: Monkey : mammals

Explanation:
Monkeys are mammals (vertebrates that give birth to a live young)
Aligator is a reptile (mostly egg-laying creatures).
Amphibians: Animals living both on land and in water. (Eg: Frog)
Aquatics: Animals which live in water (fishes, etc)
Rhodents: rats, mice, squirrels, etc.

Choose the pair of words which have a similar relationship to that between the given pair of words:

If barometer is to air pressure, then bolometer is to

a. electromagnetic radiation

b. magnetic declination

c. wind speed

d. altitude
ANSWER: electromagnetic radiation

Explanation:
Barometer is used to measure air pressure. Similarly, bolometer is used to measure electromagnetic
radiation.
The device used to measure –
magnetic declination - declinometer
wind speed – anemometer
altitude – altimeter

Choose the pair of words which have a relationship similar to that between the given pair of words:

Soldiers : Army

a. Ships : crew

b. Elephants : Herd

c. Sailors : fleet

d. Keys : Bundle

ANSWER: Elephants : Herd

Explanation:
In the given pair of words, Army is a collective noun of soldiers.
Collective noun is used to refer a group of nouns as one entity.

a) A fleet of ships
b) A herd of elephants
c) A crew of sailors
d) A bunch of keys

Choose the pair of words which have a similar relationship to that between the given pair of words:

Frog : Tadpole

a. Boar : Leveret

b. Swan : Cygnet

c. Hare : Foal

d. Goat : Cub

ANSWER: Swan : Cygnet


Explanation:
The young swans are called as swanlings or cygnets.

Young names of the animals are:

Boar : Porkling
Hare : Leveret
Goat : Kid, billy
Horse : Foal
Tiger : Cub

Choose the pair of words which have a similar relationship to that between the given pair of words:

Telephones : Buzz

a. Trumpets : Blare

b. Coins : clinks

c. Woods : chatter

d. Thunder : Creak

ANSWER: Trumpets : Blare

Explanation:
The relation given above is between the noun subjects and sounds made by them.

Coins : Jingle
Woods : Crackle
Thunder : Rumbles
Chains : Clank
Teeth : Chatter
A coin : clinks

Choose the pair of words which have a similar relationship to that between the given pair of words:

Monkeys : climb

a. Ducks : leap

b. Cats : flap

c. Rabbits : waddle

d. Horses : trot
ANSWER: Horses : trot

Explanation:
The given pair of words indicate movements of animals.
As monkeys climb the trees, horses trot (The diagonal pairs of legs move forward at same time).
Similarly,
Ducks: waddle (walk with short steps)
Cats : stalk
Rabbits : leap

Choose the pair of words which have a similar relationship to that between the given pair of words:

Stabilizer : Refrigerator

a. computer : laptop

b. axle : motorbike

c. regulator : fan

d. wheel : car

ANSWER: regulator : fan

Explanation:
Stabilizers feed constant voltage current to electrical gadgets like refrigerators and protect them from
damage due to voltage fluctuations. It is not a part of refrigerator but is used to protect it.
Option c) Similarly, regulator is not a part of fan but it only increases or decreases the speed of fan.
Option a) Computer or laptop are different and are independent.
Option b), d) axle is a part of motorbike and wheel is a part of car.
Hence, the correct option is (c)

Choose the pair of words which have a relationship similar to that between the given pair of words:

Sum: Plus

a. Subtract:Minus

b. Donkey:Cart

c. Life: Death

d. None of the above

ANSWER: Subtract:Minus
Explanation:
Just like a sum involves addition or plus sign, subtraction involves minus sign. Life is the opposite of
death and donkey is known as a means of pulling the cart. So the right choice is (a)

Choose the pair of words which have a relationship similar to that between the given pair of words:

Swing: Park

a. Chimney: House

b. Bed: Furniture

c. Bed: Room

d. None of the above

ANSWER: Bed: Room

Explanation:
A swing is a type of object found in a park whereas a bed is found in a room. The correct choice is (c).

Choose the pair of words which have a relationship similar to that between the given pair of words:

Chance: Encounter

a. Deep: Sea

b. Artist: Doctor

c. Patient:Hospital

d. Educator: Teacher

ANSWER: Deep: Sea

Explanation:
Just like “chance encounter” is a phrase, so is “deep sea”. The other word pairs are either not linked or
do not form a meaningful whole.

Choose the pair of words which have a relationship similar to that between the given pair of words:

Perspective: Viewpoint

a. Sad: Disheartened

b. Clinic: Doctor
c. Book: Pencil

ANSWER: Sad: Disheartened

Explanation:
Perspective and viewpoint are same in meaning, so is a while b and c are simply connected or associated
word pairs.

Choose the pair of words which have a relationship similar to that between the given pair of words:

Management: CEO

a. Journalism: Managing Editor

b. Office: Clerk

c. Hospital: Paramedic

d. None of the above

ANSWER: Journalism: Managing Editor

Explanation:
CEO heads the management. Similarly, the managing editor heads a team of journalists. But a clerk does
not head the office. Similarly, a paramedic is not the head of the department of the hospital’s sections.
So (a) is the correct choice.

Choose the pair of words which have a relationship similar to that between the given pair of words:

Pleasant: Dislikable

a. Sorry: Defiant

b. Guilty: Innocent

c. Happy: Overjoyed

d. Sad: Sorrowful

ANSWER: Guilty: Innocent

Explanation:
Pleasant is the opposite of dislikable for example Harish is a pleasant and happy person while Mahesh is
thoroughly dislikable. Similarly, guilty is the direct opposite of innocent and therefore the correct
answer. The other two options have word pairs which mean the same thing.

Choose the pair of words which have a relationship similar to that between the given pair of words:
Stench: Fragrance

a. Sound: Clap

b. Sound: Silence

c. Sound: Chatter

d. Sound: Noise

ANSWER: Sound: Silence

Explanation:
Silence is the opposite of sound while clap, chatter and noise are all forms of sound that can be heard.
Stench is the opposite of fragrance therefore the correct choice is (b).

Choose the pair of words which have a relationship similar to that between the given pair of words:
Healer : Doctor

a. Pilot: Airhostess

b. Artist: Designer

c. Patient:Doctor

d. Educator: Teacher

ANSWER: Educator: Teacher

Explanation:
Doctor is a type of healer while pilots and air hostesses are allied professions. Artist and designer are
similar professions and educators include professors, lecturers and teachers. Therefore a teacher is a
type of educator and (d) is the correct answer.

Choose the pair of words which have a relationship similar to that between the given pair of words,

Penniless: Millionaire

a. Arrogant: Wealthy

b. Indigenous: Native

c. Crude: Refined

d. Gorgeous: Lovely

ANSWER: Crude: Refined


Change of Voice

The given sentence is in active/ passive voice. Change the voice of the sentence. Select the correct
option from the sentence below.

He has a gut feeling that his brother may be keeping a secret.

a. He has a gut feeling that a secret may be being kept by his brother.

b. A gut feeling is had that a secret is kept by his brother.

c. He had a gut feeling that a secret was kept by his brother.

d. He has a gut feeling that maybe his brother kept a secret.

ANSWER: He has a gut feeling that a secret may be being kept by his brother.

The given sentence is in active/ passive voice. Change the voice of the sentence. Select the correct
option from the sentence below.

She has preserved all the old letters of her dead lover.

a. All the old letters of her dead lover have been preserved by her.

b. All the old letters of her dead lover has been preserved by her.

c. All the old letters have been preserved.

d. The letters of her old lover have been preserved.

ANSWER: All the old letters of her dead lover have been preserved by her.

The given sentence is in active/ passive voice. Change the voice of the sentence. Select the correct
option from the sentence below.

Champagne is drunk on new year’s eve.

a. People drink champagne on new year’s eve.

b. Let us drink champagne on new year’s eve.

c. They will drink champagne on new year’s eve.

d. People always drink champagne on new year’s eve.


ANSWER: People drink champagne on new year’s eve.

The given sentence is in active/ passive voice. Change the voice of the sentence. Select the correct
option from the sentence below.

If you’re doing something important, you are working to make change happen.

a. If something important is being done, work is being done to make change happen.

b. If something important is done, work is done to make change happen.

c. Something important is being done by you, you are making change happen.

d. If important work is being done, change is about to happen.

ANSWER: If something important is being done, work is being done to make change happen.

The given sentence is in active/ passive voice. Change the voice of the sentence. Select the correct
option from the sentence below.

He had kept up his diet program for a month.

a. His diet program had been kept up for a month.

b. His diet program was kept up for a month.

c. His diet program had been kept for a month.

d. The doctor had kept up his diet program for a month.

ANSWER: His diet program had been kept up for a month.

The given sentence is in active/ passive voice. Change the voice of the sentence. Select the correct
option from the sentence below.

I keep the butter in the fridge.

a. Fridge is where the butter has been kept.

b. Butter is kept in the fridge.

c. Butter was kept in the fridge by me.

d. They keep the butter in the fridge.


ANSWER: Butter is kept in the fridge.

The given sentence is in active/ passive voice. Change the voice of the sentence. Select the correct
option from the sentence below.

She bought a pearl necklace.

a. A pearl necklace was bought by her.

b. A pearl necklace had been bought by her.

c. A pearl necklace had been bought for her by him.

d. A pearl necklace was bought for her.

ANSWER: A pearl necklace was bought by her.

The given sentence is in active/ passive voice. Change the voice of the sentence. Select the correct
option from the sentence below.

May you be blessed with happiness.

a. I bless you with happiness.

b. May God bless you with happiness.

c. God blesses you with happiness.

d. God will bless you with happiness.


ANSWER: May God bless you with happiness.

The given sentence is in active/ passive voice. Change the voice of the sentence. Select the correct
option from the sentence below.

He kept me waiting.

a. He asked me to keep waiting.

b. I was kept waiting.

c. He asked me to wait and did not return.

d. I was waiting for him.

ANSWER: I was kept waiting.


The given sentence is in active/ passive voice. Change the voice of the sentence. Select the correct
option from the sentence below.

My bicycle has been sold.

a. I had sold my bicycle.

b. I have sold my bicycle.

c. They sold my bicycle.

d. My bicycle will sell.

ANSWER: I have sold my bicycle.

Change of Speech

The given sentence is in direct/ indirect speech. Change the narration of the sentence. Select the
correct option from the sentence below.

The young man promised her that he would be there the next day.

a. The young man said, “I will be there the next day”

b. The young man said, “I promise to be here tomorrow.”

c. The young man said, “I will try to be there the next day.”

d. The young man said, “I will be here the next day.”

ANSWER: The young man said, “I promise to be here tomorrow.”

The given sentence is in direct/ indirect speech. Change the narration of the sentence. Select the
correct option from the sentence below.

He said, ‘’I hardly have time to attend any party.”

a. He said that he has no time to attend any party.

b. He said that he hardly has time to attend any party.

c. He said that he hardly had time to attend any party.

d. He says that he hardly had time to attend any party.

ANSWER: He said that he hardly had time to attend any party.


The given sentence is in direct/ indirect speech. Change the narration of the sentence. Select the
correct option from the sentence below.

The young knight enquired which way the princess had gone.

a. The young knight asked, “Which way has she gone?”

b. The young knight asked, “Which way did the princess go?”

c. The young asked the old man where the princess had gone.

d. The young knight enquired, “Which way the princess went?”

ANSWER: The young knight asked, “Which way did the princess go?”

The given sentence is in direct/ indirect speech. Change the narration of the sentence. Select the
correct option from the sentence below.

He said, ‘’I have written a prize-winning essay.”

a. He said that he has written a prize-winning essay.

b. He said that I have written a prize-winning essay.

c. He said that he had written a prize-winning essay.

d. He says that he has written a prize-winning essay.

ANSWER: He said that he had written a prize-winning essay.

The given sentence is in direct/indirect speech. Change the narration. Choose the correct answer from
the alternatives below.

The mother will say, “ My daughter is going to school”.

a. The mother will say that her daughter is going to school.

b. The mother will say that her daughter will be going to school.

c. The mother would be saying that her daughter would be going to school.

d. The mother will say that her daughter will go to school.

ANSWER: The mother will say that her daughter is going to school.

The given sentence is in direct/indirect speech. Change the narration. Choose the correct answer from
the alternatives below.
Hari said to me, ‘’I will go to the market tomorrow’’.

a. Hari told me that he will go to the market.

b. Hari told me that he will go to the market tomorrow.

c. Hari told me that he would go to the market tomorrow.

d. Hari told me that he would go to the market the next day.

ANSWER: Hari told me that he would go to the market the next day.

The given sentence is in direct/indirect speech. Change the narration. Choose the correct answer from
the alternatives below.

He requested her to accompany him.

a. He said to her, “Please, come with me.”

b. He says, “Please come with me.”

c. He says, “Let her come with me.”

d. He said, “You have to come with me.”

ANSWER: He said to her, “Please, come with me.”

The given sentence is in direct/indirect speech. Change the narration. Choose the correct answer from
the alternatives below.

“I will avenge your murder,” she cried over her husband’s corpse.

a. She decided to avenge her husband’s death.

b. She decided to avenge his murder.

c. She cried over her husband’s corpse that she would revenge his murder.

d. She cried over her husband’s corpse that she would avenge his murder.

ANSWER: She cried over her husband’s corpse that she would avenge his murder.

The given sentence is in direct/indirect speech. Change the narration. Choose the correct answer from
the alternatives below.
He says he is unwell.

a. a) He said, “I am unwell.”

b. b) He says ,”I was unwell.”

c. c) He says, “He was unwell.”

d. d) He said, “I were unwell.”

ANSWER: a) He said, “I am unwell.”

The given sentence is in direct/indirect speech. Change the narration. Choose the correct answer from
the alternatives below.

He said to me, “ I don’t believe you.”

a. He decided not to believe me.

b. “I don’t believe you’’ he said.

c. He said he not believe me.

d. He told me that he didn’t believe me.

ANSWER: He told me that he didn’t believe me.

You might also like